Power Generation from Constant Rotating Mass

Click For Summary
A rotating mass cannot generate electrical power without an additional system to convert mechanical energy into electrical energy. The discussion centers on calculating the power output of a rotating object using the formula P = dW/dt, where work done (W) is related to the torque and angular velocity. For a mass of 60 kg, radius of 5 meters, and speed of 10 meters per second, the specifics of the conversion process are crucial. The conversation emphasizes the need to understand the relationship between mechanical motion and electrical energy generation. Ultimately, without a conversion mechanism, the rotating mass alone does not produce electrical power.
Hanababa
Messages
11
Reaction score
0
An object with a mass M is rotating around an axis, with an arm of radius R at a certain speed V.
Assuming that the movement is constant, and a 100% efficiency, how many watt of electrical power can this device generate ?

For example purpose, we can take M=60 kg, R=5 meters and V=10 meters per second.

I can't find all the formulas and am lost with the units.

Thanks a lot
 
Physics news on Phys.org
That device can't generate any electrical power.
 
Hello!

I assume it is some kind of engine, and rotary motion somehow generates electricity through another undisclosed system.

I would try to calculate how much Power our system has at any point, and if this motion is constant and translates to Electrical Energy - this is the generated power.

Try to find the work done by the system and how much time it takes to preform (lets say for one revolution) and then you can find the power of the system: P = dW/dt

(useful equation: P(t) = torque(t) * angular_velocity(t) )
 
Last edited:
Question: A clock's minute hand has length 4 and its hour hand has length 3. What is the distance between the tips at the moment when it is increasing most rapidly?(Putnam Exam Question) Answer: Making assumption that both the hands moves at constant angular velocities, the answer is ## \sqrt{7} .## But don't you think this assumption is somewhat doubtful and wrong?

Similar threads

  • · Replies 74 ·
3
Replies
74
Views
8K
  • · Replies 5 ·
Replies
5
Views
3K
  • · Replies 5 ·
Replies
5
Views
3K
Replies
12
Views
3K
  • · Replies 32 ·
2
Replies
32
Views
6K
  • · Replies 41 ·
2
Replies
41
Views
4K
Replies
7
Views
2K
Replies
6
Views
2K
  • · Replies 4 ·
Replies
4
Views
1K
Replies
8
Views
3K